Emergency Medicine Pediatric CAQ

Ace your homework & exams now with Quizwiz!

Question: What is the dose of dexamethasone for croup in children?

Answer: 0.6 mg/kg (up to 10 mg maximum dosage) typically given orally.

Question: In addition to racemic epinephrine, what other medication is often used in the treatment of croup?

Answer: A corticosteroid, such as dexamethasone, is often given to help reduce airway inflammation

Question: What type of bronchoscope should be used to remove a foreign body?

Answer: A rigid bronchoscope is used to remove foreign bodies, whereas flexible bronchoscopy can be helpful for diagnosis in equivocal cases.

Question: What is the initial diagnostic imaging of choice in suspected acute appendicitis in children?

Answer: Abdominal ultrasound.

Question: What is the leading cause of cardiac death among the young in developing countries?

Answer: Acute rheumatic fever.

Question: What is the most common cause of conjunctivitis in school-age children?

Answer: Adenovirus.

Question: What is the first-line treatment for intussusception?

Answer: Air enema.

Question: What medication (and dose) can be used to temporarily preserve patency of the ductus arteriosus?

Answer: Alprostadil (prostaglandin E1), 0.05-0.1 mcg/kg/min intravenously.

Question: What ethnic group in the US has the highest incidence of Kawasaki disease?

Answer: Americans of Asian and Pacific island descent.

Question: What is the most common cause of painful rectal bleeding in infants?

Answer: Anal fissure.

Question: Which branch of the median nerve is commonly injured in supracondylar fractures?

Answer: Anterior interosseous. The integrity is checked by strength of patient making an "OK" sign.

Question: What percentage of infants with necrotizing enterocolitis are born at full term?

Answer: Approximately 10%.

Question: What is the treatment for a hemodynamically unstable child with supraventricular tachycardia?

Answer: Cardioversion at 0.5 to 1 J/kg. If unsuccessful, increase energy dose to 2 J/kg on subsequent attempts.

Question: What is the classic appearance of midgut volvulus on upper GI series?

Answer: Corkscrew appearance.

Question: Duodenal atresia is associated with which congenital syndrome?

Answer: Down syndrome.

Question: Classically, how many g/dL of deoxygenated blood is required before cyanosis is apparent?

Answer: Five g/dL.

Question: How many days after antibiotic administration will a patient with pertussis remain infectious?

Answer: For 5 days.

Question: If a child has a foreign body in the left mainstem bronchus, what would you expect to see on left lateral decubitus radiograph of the chest?

Answer: Hyperinflation of the left lung.

Question: What is the most immediate life-threatening electrolyte abnormality in tumor lysis syndrome?

Answer: Hyperkalemia due to its propensity to cause fatal dysrhythmias.

Question: What is the most common concerning complication of bronchiolitis?

Answer: Infants infected with RSV (particularly premature infants) are at increased risk for developing apnea.

Question: What viral infection is known to greatly increase the susceptibility to bacterial tracheitis?

Answer: Influenza A.

Question: What is the appropriate treatment of neonatal conjunctivitis caused by herpes simplex virus?

Answer: Intravenous acyclovir.

Question: Should frog leg views be obtained to evaluate for SCFE?

Answer: It is controversial, given the potential for further epiphyseal displacement in this position.

Question: What is another name for croup?

Answer: Laryngotracheobronchitis.

Question: Which sex is most commonly diagnosed with pyloric stenosis?

Answer: Males (4 to 1 predominance).

Question: What organisms can cause both pneumonia and vesicular inflammation of the tympanic membrane?

Answer: Mycoplasma pneumoniae and Streptococcus pneumoniae.

Question: What class of medication is oseltamivir?

Answer: Neuroaminidase inhibitor.

Question: What is the normal respiratory rate of newborns and 1-year-olds?

Answer: Newborns 50 breaths per minute, 1-year-olds 30 breaths per minute.

Question: Can staphylococcal scalded skin syndrome be spread to other body parts by rupture of the bullae?

Answer: No, the fluid inside the bullae is sterile, while the toxin is produced at a remote site and delivered to the skin via the bloodstream.

Question: What is the most common radiographic finding for a Salter-Harris Type V fracture?

Answer: Normal.

Question: Which virus is the most common cause of croup?

Answer: Parainfluenza virus.

Question: What is the appropriate first-line agent for treatment of status epilepticus in neonates?

Answer: Phenobarbital.

Question: What is the greatest risk factor for developing necrotizing enterocolitis?

Answer: Prematurity.

Question: What is the most common type of tracheoesophageal fistula?

Answer: Proximal esophageal atresia with a distal fistula connecting the esophagus to the trachea.

Question: What organism is associated with conjunctivitis and corneal ulcers in contact lens wearers?

Answer: Pseudomonas aeruginosa.

Question: What is the appropriate medication for seizures in children who overdose on isoniazid?

Answer: Pyridoxine.

Question: How is Hirschsprung's disease definitively diagnosed?

Answer: Rectal biopsy.

Question: What is the preferred route of medication administration if intravenous or intramuscular access is not available for a seizing child?

Answer: Rectal; diazepam is the agent of choice for this purpose.

Question: In patients with TOF prone to "tet" spells, what are the classic EKG findings?

Answer: Right ventricular hypertrophy (dominant R wave in the right precordium) and right axis deviation.

Question: What pediatric injury is represented by objective neurologic signs resulting from trauma without evidence of injury on plain radiographs or computed tomography?

Answer: SCIWORA (Spinal Cord Injury Without Radiographic Abnormality).

Question: What other mechanism can retinal hemorrhages occur in other than abusive head trauma?

Answer: Severe blunt force mechanism such as a motor vehicle collision or fall from a great height.

Question: What is the most common cause of meningitis in adult patients?

Answer: Streptococcus pneumoniae.

Question: Where is the narrowest area of the pediatric airway?

Answer: The cricoid ring (as opposed to the vocal cords in adults).

Question: In what age are airway obstructions most common?

Answer: The mean age is 3.5 years old.

Question: What is the Thurstan Holland sign?

Answer: The triangular metaphyseal fragment in the metaphysis from a Salter-Harris Type II fracture.

Question: What is the X-ray finding of epiglottitis?

Answer: Thumbprint sign.

Question: What is the most common cause of acute onset childhood hip pain?

Answer: Transient synovitis.

Question: What causes desaturation on pulse oximetry in asthmatic patients after administering albuterol without oxygen?

Answer: V/Q mismatch; ß2 agonists cause pulmonary vasodilation increasing perfusion to poorly ventilated lung units thus worsening hypoxemia initially.

Question: What is the second most common cause of chronic stridor in infants?

Answer: Vocal cord paralysis.

A 3-year-old girl presents to the ED with 1 hour of a barking cough and inspiratory stridor at rest. On exam, she has mild retractions but is not hypoxic. Which of the following interventions has been shown to reduce hospital length of stay in moderate to severe croup? Dexamethasone Heliox Humidified air Racemic epinephrine

Correct Answer ( A ) Explanation: Glucocorticoids are considered the mainstay of treatment for moderate to severe croup. They are the only treatment that alters the natural course of the illness. Dexamethasone has been shown to reduce the use of epinephrine, severity scores at 6 hours, hospital lengths of stay, and readmissions. In some studies, it has also reduced the rate of intubation. Neither heliox (B) nor humidified air (C) has shown any benefit in the treatment of croup. Racemic epinephrine (D) works acutely, likely by causing local vasoconstriction and reducing hyperemia and edema. It has a very transient effect, however.

What is the most common causative organism of meningitis in a 1-week-old child? Group B Streptococci Haemophilus influenzae Listeria monocytogenes Staphylococcus aureus

Correct Answer ( A ) Explanation: Group B Streptococci (GBS) is the most common causative pathogen for meningitis in neonates. There are many bacteria that can cause meningitis. The most likely causative organism changes based on host factors including age, comorbidities and immune status. It is important to know the most common causes in different patient groups in order to tailor antibiotic regimens. In neonates, group B streptococci account for more than 75% of cases of meningitis. Other common organisms include Streptococcus pneumoniae, Neisseria meningitides and Listeria monocytogenes. GBS is transmitted from the mother during childbirth. As such, rates of GBS have fallen with the wide spread practice of intrapartum prophylaxis. Neonates with meningitis will present nonspecifically with behavioral changes, neurologic changes and vital sign abnormalities. Because of the nonspecific presentations all neonates with a fever should be considered for lumbar puncture to rule out the diagnosis.

6-year-old boy with a history of asthma presents to the ED with an acute asthma exacerbation in the setting of an upper respiratory tract infection. He looks well and is breathing comfortably though he is wheezing. His mother says she has been unable to pay for his prescribed medications. In addition to administering inhaled bronchodilators, you decide to begin the patient on a course of oral steroids. Which of the following is a reason to give oral dexamethasone as opposed to oral prednisolone? Dexamethasone has a higher rate of adherence Dexamethasone has greater efficacy in children with a history of asthma Prednisolone has a lower initial dose Prednisolone has a shorter medication course

Correct Answer ( A ) Explanation: Prednisolone commonly causes emesis in children. In acute asthma exacerbations, it is also given as a 5-day course; dexamethasone is a 2-day course. Given its lower frequency of emesis and its shorter course, dexamethasone is often preferred in patients who have not tolerated steroids in the past or are prone to poor medication adherence for a variety of reasons. Dexamethasone has been shown to be at least as effective as but not superior (B) to prednisolone. The dose of prednisolone (1 mg/kg PO) (C) is higher than that of dexamethasone (0.6 mg/kg PO). As discussed above, prednisolone has a longer (D) medication course.

12-year-old boy presents to the ED with left thigh pain that radiates to the knee. He states that he tripped and fell while walking tonight. His mother reports that he has been limping for the past couple of weeks. On exam, the patient cries with any attempt at rotating his left hip. An X-ray is obtained, as seen above. Which of the following statements is true regarding his condition? Avascular necrosis is a common complication Most cases can be managed as an outpatient with partial weightbearing and crutches Most commonly caused by blunt trauma The typical patient is between 6 and 8 years of age

Correct Answer ( A ) Explanation: Slipped capital femoral epiphysis (SCFE) is the most common hip disorder in adolescents and typically affects boys more often than girls in a 2:1 ratio with peak incidence of 13 and 11 years of age, respectively. The etiology is believed to result from physeal cartilage weakness associated with the onset of puberty and growth spurt but may be multifactorial. The typical patient is an obese adolescent male with hip, thigh, or isolated knee pain. The diagnosis is usually made by visualizing the bony abnormality on AP and lateral hip X-rays. The initial finding is asymmetry of the femoral epiphysis in relation to the neck. This is often referred to as "the scoop slipping off the ice cream cone." Another method is to draw a line along the lateral border of the femoral neck (Klein's line) that should intersect some part of the femoral head. Failure of the line to intersect the head indicates that there is medial and posterior movement of the head in relation to the femoral neck. Complications include avascular necrosis of the hip and premature closure of the physis. SCFE is the most common cause of hip disability in adolescents. Management includes strict nonweightbearing (B) and definitive operative management. Consequences of a missed diagnosis may lead to avascular necrosis of the femoral head and need for hip replacement. Prophylactic pinning of the contralateral hip is often done because of the high association of SCFE in the contralateral hip within 18 months. It often presents insidiously (C) and may occur after minimal trauma. It may take weeks or months before a diagnosis is made and is often times missed initially. Legg-Calvé-Perthes disease most commonly occurs in individuals aged 6 to 8 years (D).

Which of the following is recommended by the American Academy of Pediatrics Task Force on sudden unexpected death of infancy to reduce the risk of sudden unexpected death of infancy in the general population? Parents should consider offering a pacifier at nap time and bedtime Parents should place infants in the prone position for sleep Parents should place infants to sleep on soft mattresses Parents should use home monitoring systems

Correct Answer ( A ) Explanation: Sudden unexpected infant death (SUID) is defined as "the sudden death of an infant less than one year of age, which remains unexplained after a thorough case investigation, including performance of a complete autopsy, examination of the death scene, and review of clinical history." It is the third leading cause of death in infants and may occur at any time during the first two years of life, but it is rare in children younger than one month of age and in those older than one year of age. Most cases occur between two and four months of age. The American Academy of Pediatrics Task Force on sudden unexpected death of infancy recently published guidelines to help reduce the risk of SUID in the general population.

12-year-old boy is brought by his father to the Emergency Department with a one day history of abdominal pain associated with malaise and loss of appetite. HIs abdominal pain started in the periumbilical area and later localized in the right lower quadrant. Physical examination reveals an uncomfortable febrile child with clear breath sounds, tachycardia, direct and rebound tenderness at the right lower quadrant, capillary return time of less than 2 seconds, and full pulses. Which of the following symptoms is most consistent with this boy's likely diagnosis? Anorexia Body malaise Diarrhea Fever

Correct Answer ( A ) Explanation: The child presents with the common manifestations of acute appendicitis. Appendicitis starts with the obstruction of the appendiceal lumen by fecal material, lymphoid hyperplasia, parasites, or tumors. Obstruction increases the intraluminal pressure within the appendix leading to lymphatic and venous congestion and edema which then impairs arterial perfusion. When there is no blood perfusing, ischemia ensues, and further bacterial invasion of the appendiceal wall results in necrosis. This process correlates with the clinical manifestations and progression from simple appendicitis to gangrenous appendicitis, and ultimately to appendiceal perforation. The classic presentation of acute appendicitis involves temporal progression of symptoms from vague mild abdominal pain, body malaise, and anorexia to severe localized abdominal pain, fever, and vomiting which typically occur within 24 to 48 hours. Abdominal pain is the first symptom characterized as initially vague and colicky located in the periumbilical area which later localizes in the right lower quadrant. Anorexia is a classic and consistent finding in acute appendicitis occurring in up to three-quarters of patients. Absence of anorexia makes the diagnosis of acute appendicitis less likely.

A 3-week-old boy is brought in by parents for a worsening cough and nasal discharge for the past week. Although he is afebrile and appears well between coughing episodes, mom states he began to gag, turn blue, and appeared as if he could not breathe during one of his coughing episodes. A CBC showed a white blood cell count of 35 000/µL with an absolute lymphocyte count of 17,000/µL. In addition to admitting the patient to the pediatric ICU and sending a polymerase chain reaction test to confirm the diagnosis, which of the following is the antibiotic of choice for an infant this age? Azithromycin Clarithromycin Erythromycin Trimethoprim-sulfamethoxazole

Correct Answer ( A ) Explanation: The patient's presentation is classic for pertussis, commonly known as whooping cough. Pertussis is highly contagious, developing in approximately 80%-90% of susceptible individuals who are exposed to it. Most cases occur in the late summer and early fall. Pertussis is most severe for infants under 6 months, where the mortality rate is approximately 1%; more than 50% of these deaths occur in infants <2 months, prior to their first set of immunizations. Infants <6 months of age may present with gasping, gagging, and apnea rather than the classic whoop. Leukocytosis with absolute lymphocytosis occurs during the late catarrhal and paroxysmal phases. It is a nonspecific finding but correlates with the severity of the disease. The goal of antimicrobial therapy is to decrease the severity of symptoms in the individual as well as decrease the spread of infection to close contacts. Once the paroxysmal phase has started, antibiotics are not effective in altering the course of illness, given that the illness is due to toxin-mediated effects. It does, however, shorten the period of contagiousness. Azithromycin is the first-line treatment for pertussis in the youngest patients, although it is associated with a slightly increased risk of infantile hypertrophic pyloric stenosis. Clarithromycin (B) and erythromycin (C) are not recommended for infants less than 1 month because their use has been associated with increased risk of infantile hypertrophic pyloric stenosis. Erythromycin was for many years, the standard regimen for the treatment of pertussis. However, the 2012 Report of the American Academy of Pediatrics (AAP) Committee on Infectious Diseases and CDC prefer the use of azithromycin for infants younger than 1 month of age. Azithromycin is associated with fewer adverse gastrointestinal events, may be dosed once daily, and does not inhibit the cytochrome P450 system and thus is preferable to erythromycin. Trimethoprim-sulfamethoxazole (D) is contraindicated in infants less than 2 months. Patients who are aged 2 months or older with hypersensitivity to macrolides may be treated with trimethoprim-sulfamethoxazole.

A 4-day-old boy born at home presents to the ED with constipation. Parents report that he has not passed any stool since birth. On exam, the child has a distended abdomen and cries when it is palpated. Which of the following is the most likely diagnosis? Hirschsprung disease Intussusception Midgut volvulus Pyloric stenosis

Correct Answer ( A ) Explanation: This child is presenting with signs and symptoms associated with Hirschsprung disease, also known as congenital aganglionic megacolon. Hirschsprung disease is caused by the lack of ganglion cells in the colon, leading to increased muscle tone and contractility of the aganglionic segment. The majority (75%) occur in the rectosigmoid segment. The most common presenting complaint is delayed passage of meconium, greater than 48 hours after birth. Other signs and symptoms include diminished stool frequency, abdominal distension, and bilious emesis (late finding). Complications of Hirschsprung disease include enterocolitis and toxic megacolon, which in this disease process would present as massive dilation of the colon proximal to the aganglionic segment. This leads to bloody stool, fever, and high risk of perforation. Hirschsprung disease may be diagnosed with barium enema (reveals a cone-shaped transitional zone), rectal manometry (reveals lack of anal sphincter relaxation), or definitively on rectal biopsy. Initial management includes gastric decompression, a rectal tube, and intravenous fluids. Definitive management is via resection of the aganglionic segment.

9-year-old boy is brought in by his mom for bizarre behavior. She states that he has had intermittent fevers for the past week and is now complaining of joint pain and swelling. Most concerning for Mom is the writhing, purposeless, and uncontrollable movements of her son's hands that she observed this morning. On exam, a diastolic murmur is noted at the right upper sternal border. Which of the following tests is most likely to confirm the diagnosis? Antistreptolysin O titer (ASO) CBC ECG Echocardiogram Myocardial biopsy

Correct Answer ( A ) Explanation: This patient has acute rheumatic fever (ARF), a systemic disease triggered by a complex hyperimmune response to group A streptococcal (GAS) pharyngitis. The American Heart Association has developed and modified the Jones criteria to enable physicians to identify patients with ARF. To fulfill the criteria, the patient must exhibit evidence of recent streptococcal disease and have either two major criteria or one major and two minor criteria. This patient has three of the major criteria: chorea (writhing, purposeless, and uncontrollable movements of the hands); carditis (diastolic murmur suggestive of aortic insufficiency); and arthritis. Evidence of recent streptococcal infection is required to confirm the diagnosis. This can be accomplished by detecting an elevated or rising ASO titer or by obtaining a positive throat culture or rapid Strep antigen test. Thrombocytosis in a CBC (B) may be seen in ARF because platelets are an acute phase reactant. However, this will not confirm the diagnosis of ARF. Thrombocytosis is also seen in Kawasaki syndrome. An ECG (C) may be abnormal in this patient, possibly showing evidence of strain due to underlying aortic insufficiency, but would not confirm the diagnosis. At some point, this patient will require an echocardiogram (D), but this will not aid in determining or confirming the underlying diagnosis of ARF. Myocardial biopsy (E) is an invasive procedure that is not required to make the diagnosis of acute rheumatic fever. It is sometimes used in the diagnosis of myocarditis.

5-week-old male infant presents to the ED with nonbilious emesis. Per parents, the patient seems hungry, but 5-10 minutes after drinking 3 ounces of formula, he spits up. This was initially sporadic, but it now occurs with every feeding. His mother describes the vomiting as forceful. On exam, the patient appears well hydrated with a temperature of 37.4°C, pulse of 150 bpm, and respiratory rate of 30/min. His abdomen is nondistended with no masses or palpable organomegaly. The patient has a strong suck and symmetrical Moro reflex. Which of the following diagnostic tests should be ordered to confirm the diagnosis? Abdominal ultrasound Abdominopelvic CT scan Basic metabolic panel Nasogastric tube insertion Upper GI series

Correct Answer ( A ) Explanation: This patient has pyloric stenosis, the most common cause of gastric obstruction in infants. It is characterized by progressive postprandial, nonbilious vomiting that steadily increases in frequency and amount due to hypertrophy of the pyloric musculature and edema of the pyloric canal, producing gastric outlet obstruction. Infants typically present with symptoms at 2-8 weeks, but they may occur up to 5 months of age. Due to persistent vomiting, hypochloremic hypokalemic metabolic alkalosis with dehydration is commonly noted. Failure to thrive may also be seen if the diagnosis is not made early in the course. Up to 90% of infants exhibit the pathognomonic pyloric olive, which is due to hypertrophy of the antral and pyloric musculature and is best palpated in the epigastrium or right upper quadrant. If a palpable mass is detected, no imaging is indicated and direct surgical consultation is warranted. However, if the diagnosis is less certain and there is no palpable mass detected, an ultrasound should be obtained.

9-day-old girl presents with to the ED with unilateral eye drainage, redness and conjunctival edema. Which of the following is the most likely etiology of this patient's presentation? Chlamydia trachomatis Herpes simplex virus Neisseria gonorrhoeae Respiratory syncytial virus

Correct Answer ( A ) Explanation: This patient is presenting with signs and symptoms consistent with chlamydial neonatal conjunctivitis. This is caused by the organism Chlamydia trachomatis. Neonates contract chlamydial conjunctivitis via exposure during vaginal delivery. Silver nitrate and erythromycin ointment are commonly used in neonates for prophylaxis against neonatal conjunctivitis; however, these agents are less effective against chlamydia than gonorrhea. Chlamydial neonatal conjunctivitis typically presents 5 to 14 days after birth. Signs and symptoms consist of classically unilateral ocular hyperemia, conjunctival edema, and watery or purulent ocular discharge. There is less systemic illness than is seen with Neisseria gonorrhoeae. The diagnosis is confirmed with cell scraping from the palpebral conjunctiva. Management of chlamydial neonatal conjunctivitis is with an oral macrolide, such as erythromycin. These patients may frequently be treated as outpatients; however, treatment must be strictly adhered to as chronic chlamydial conjunctivitis and scarring is a frequent cause of blindness worldwide. Herpes simplex virus (B) can cause neonatal conjunctivitis. It typically presents within 2 to 14 days after birth. It presents with an injected, red, painful eye, and the diagnosis is confirmed by the presence of epithelial dendrites on slit lamp exam. Neisseria gonorrhoeae (C) is a common cause of neonatal conjunctivitis, but it typically presents 2 to 4 days after birth with copious bilateral purulent ocular discharge and systemic illness. Respiratory syncytial virus (D) is a common cause of viral conjunctivitis in children outside the neonatal period.

Which of the following is the most common manifestation of abusive head trauma in infants? Epidural hematoma Retinal hemorrhage Subarachnoid hemorrhage Subdural hematoma

Correct Answer ( B ) Explanation: Abusive head trauma (previously referred to as shaken baby syndrome) is a form of inflicted head trauma and the leading cause of child abuse fatalities. It is a well-recognized clinical syndrome caused by violent shaking of infants, direct blows to the head, dropping or throwing a child, and asphyxia. It typical occurs in infants younger than 1 year, but may be seen in children up to 3 years old. The classic constellation of abusive head trauma includes subdural hematoma, traumatic brain injury, and retinal hemorrhages. Retinal hemorrhages are present in up to 75% of cases and have a high specificity. Absence of retinal hemorrhage does not rule out child abuse. The pathophysiology of retinal hemorrhages is uncertain. It is unclear whether bleeding is a result of increased intracranial pressure transmitted to the eye or occurs directly within the eye itself, perhaps through increased pressure along the retinal vein with subsequent disruption of the vessel. Retinal hemorrhages may involve the area in front of the retina (preretinal hemorrhages), the vitreous, and the subretinal space in addition to the retina. Hemorrhages may be described as "dot and blot" hemorrhages or flame or splinter hemorrhages.

A two-year-old child is seen in the emergency room with recent onset of cough and progressive stridor. Respiratory cultures are taken, and you are concerned for bacterial tracheitis. What is the most common organism isolated on respiratory cultures in bacterial tracheitis? Haemophilus influenzae type B Staphylococcus aureus Streptococcus pneumoniae Streptococcus pyogenes

Correct Answer ( B ) Explanation: Bacterial tracheitis can be seen in healthy children between three months and six years of age with a peak occurrence between three and five years old. Parents will often describe URI symptoms prior to bringing their child in to be evaluated. Blood cultures rarely are positive, but respiratory cultures are commonly polymicrobial with staphylococcus aureus the most common isolated species. Bacterial tracheitis is usually diagnosed on a clinical basis. The classic presentation is a child with suspected croup who continues to worsen despite treatment. High fever develops. These patients are at increased risk for airway compromise. They should be immediately started on broad-spectrum intravenous antibiotics.

Parents bring their full-term 3-week-old son to the ED after multiple episodes of bilious vomiting. On exam, you note abdominal distention and a palpable mass. Which of the following is the most likely diagnosis? Intussusception Malrotation with midgut volvulus Necrotizing enterocolitis Pyloric stenosis

Correct Answer ( B ) Explanation: During the first 3 months of gestation, there is normally a 270-degree counterclockwise rotation of the midgut, with subsequent fixation of the small bowel in the left upper quadrant (LUQ) and right lower quadrant (RLQ). When malrotation occurs, the small bowel is not anchored in the LUQ or RLQ, increasing the risk of obstruction and vascular compromise after birth. Of patients with malrotation, 33% have symptoms within the first week of life, 50% present within the first month, and 85% present within the first year. Classically, the patient will present with bilious vomiting, abdominal distension, tenderness, and a palpable mass. If not treated promptly, gangrene will occur. An abdominal plain film may show dilatation of the stomach and duodenum and a paucity of bowel gas distally. An upper GI series or ultrasound will confirm the diagnosis. Treatment is with NG tube decompression and laparotomy. Intussusception (A) is the most common cause of intestinal obstruction in children 3 months to 5 years of age. These patients typically present with intermittent paroxysms of colicky abdominal pain, vomiting, and, infrequently, currant jelly stools. Unexplained altered mental status is also associated with intussusception. Occasionally a mass in the RUQ is palpated. Necrotizing enterocolitis (C) is primarily a disease of premature infants with low birth weights. It typically occurs in the first 2 weeks of life and presents with bilious emesis, abdominal wall pain and distension, and grossly bloody stool. There are variable degrees of mucosal or transmucosal necrosis of the bowel. Pyloric stenosis (D) occurs around 2 to 6 weeks, particularly in firstborn males. A positive family history is common. It is characterized by nonbilious projectile vomiting following feeding, after which the infant will usually hungrily refeed. Occasionally, you can palpate an olive-shaped mass in the RUQ, which is the hypertrophied pylorus

Which of the following is a mainstay for the treatment of Kawasaki disease during the acute febrile phase? Aspirin - low dose Intravenous immunoglobulin (IVIG) Steroids Warfarin

Correct Answer ( B ) Explanation: Kawasaki disease is an acute illness with fever, rash, conjunctivitis, swollen lymph nodes and widespread inflammation of muscular arteries. Intravenous immunoglobulin (IVIG), as well as high dose aspirin, form the two components of the treatment of Kawasaki disease during the acute febrile phase of the disease. IVIG involves an infusion of 2 g/kg over 10 to 12 hours. High dose aspirin (80-100mg/kg/day divided into 6 hour dosing regimens) is continued until the child is afebrile for 48 to 72 hours or longer. Low dose aspirin (A) (3 to 5 mg/kg/day) is then initiated and continued until the patient shows no coronary changes by 6 to 8 weeks. The benefit of steroids (C) is controversial and current guidelines recommend restricting steroid treatment to children in whom >2 infusions of IVIG have been ineffective in alleviating fever and acute inflammation. When a patient with Kawasaki disease develops coronary artery disease, treatment may involve antiplatelet therapies (aspirin, with or without dipyridamole or clopidogrel), anticoagulant therapy with warfarin (D) or low-molecular-weight heparin, or a combination of anticoagulant and antiplatelet therapy (usually warfarin plus aspirin). No prospective data exist to guide clinicians in regards to the optimal regimen.

Which of the following is most likely to present as a ductal-dependent cardiac lesion? Atrial septal defect Coarctation of the aorta Isolated ventricular septal defect Mitral valve prolapse

Correct Answer ( B ) Explanation: Patients with coarctation of the aorta may present with circulatory failure and shock upon closure of a patent ductus arteriosus (PDA). In many cases, the coarctation of the aorta occurs juxtaductal (adjacent to the PDA). The PDA may serve to widen the juxtaductal area of the aorta so that blood may flow forward from the left ventricle. However, in other cases, the PDA serves as a conduit for right-to-left shunted blood from the right ventricle. In the latter case, infants classically present with differential cyanosis due to well-oxygenated blood reaching the upper body (pink) from the ascending aorta, and deoxygenated blood reaching the lower body (blue) via the PDA and descending aorta. This is because the PDA often inserts distal to the origin of the left subclavian artery from the aorta. When the ductus arteriosus closes, this can lead to circulatory failure and shock. Atrial septal defects (A) and isolated ventricular septal defects (C) typically feature left-to-right shunting of blood and do not require PDAs for pulmonic or systemic circulation. Although mitral valve prolapse (D) increases one's risk for cardiovascular complications in adulthood, it is not progressive in childhood and requires no specific therapy.

The pediatric glottic opening is which of the following compared to adults? Anterior, inferior Anterior, superior Posterior, inferior Posterior, superior

Correct Answer ( B ) Explanation: The glottic opening in infants is at the level of the C1 vertebra and transitions downward to C3-C4 by 7 years of age and C5 by adulthood. It is also located more anteriorly. These differences are most prominent under the age of 2. By age 8, the location of the pediatric airway is similar to that in adults.

A 1-year-old boy presents with the rhythm seen above. Physical exam is unremarkable except for tachycardia. Which of the following vagal maneuvers is the most effective to treat this rhythm? Apply external ocular pressure Apply ice to the face Blow on the patient's face with a straw Perform carotid massage

Correct Answer ( B ) Explanation: The most common dysrhythmia in children is paroxysmal supraventricular tachycardia (PSVT). It is differentiated from sinus tachycardia by its abrupt onset, rates >230, the absence of normal P waves, or by little rate variation during stressful activities. Symptoms of PSVT in infants include poor feeding, tachypnea, and irritability. Application of ice to the face has been shown to be an effective method of converting a hemodynamically stable child in SVT to NSR. It is important that when performing this maneuver that you do not occlude the nose or mouth, and apply the ice only over the patient's forehead, eyes, and bridge of the nose for 10-15 seconds. Adenosine, an AV-nodal blocking agent, can also be administered in stable patients.Applying external ocular pressure (A) is contraindicated in children because it can lead to globe rupture. Having a patient blow on an occluded straw may be an acceptable vagal maneuver, but blowing on the patient's face with a straw (C) has no benefit. Carotid massage (D) is not recommended in infants or children.

2-year-old boy is brought in to the ED by dad for a 2-day history of coughing, posttussive emesis, and diminished food intake, although he has been taking liquids. The coughing began after eating some popcorn. Mom reports that he has no fever or rhinorrhea. He had bronchiolitis at 3 months of age, but has been otherwise healthy. In the ED, his vital signs include T 37°C, HR 100 beats per min, RR 24, and pulse oximetry of 94%. On exam, he has mild nasal flaring, diffuse rhonchi, and wheezing louder on the left side of the chest. Which of the following is the best modality to confirm the diagnosis? Arterial blood gas test Bronchoscopy CT scan of the chest Inspiratory and expiratory chest radiographs

Correct Answer ( B ) Explanation: The patient in this vignette has a foreign body aspiration. It is the most common cause of accidental death in children under 1 year of age, and the risk remains very high up to 3 years of age. Peanuts, seeds, and beans are commonly inhaled items, as are small toys. All patients with a history of choking with persistent symptoms or with abnormal physical examinations such as tachypnea, retractions, wheezing, stridor, decreased unilateral breath sounds, or cyanosis should be evaluated for foreign body aspiration. Bronchoscopy is the gold standard for both diagnosis and management. The coughing history after eating popcorn suggests choking or aspiration; the persistence of symptoms warrants a bronchoscopy. Arterial blood gas (A) is helpful in determining whether the patient is adequately oxygenating and ventilating, but it will not help determine the presence of a foreign body. A CT scan (C) is of little value in an acute foreign body aspiration. It is useful when the object has been present for a period of time and has induced granulation to form around the foreign body, giving more information as to the size, location, and degree of granulation prior to extraction. Because only 20% of FBs are radiopaque, chest radiographs (D) are used only to evaluate for indirect signs of foreign body aspiration, such as unilateral hyperinflation, atelectasis, mediastinal shift, or pneumomediastinum. Expiratory chest radiographs are more sensitive than are inspiratory chest radiographs in detecting foreign body aspiration, but up to 35% of chest radiographs are falsely negative. Therefore, if the suspicion for foreign body aspiration is high, the next appropriate step is for bronchoscopy, not chest radiograph.

A 3-year-old boy presents with stridor. His mother states that he was eating a grape and suddenly started choking. The patient has normal vital signs except for an increased respiratory rate. Physical examination reveals an anxious child who is able to speak, but with stridor. Which of the following management is most likely indicated? Back blows Emergent ENT consultation Heimlich maneuver with patient supine Needle cricothyrotomy

Correct Answer ( B ) Explanation: The patient presents with a partial airway obstruction and should be kept still while ENT is consulted for emergent removal, preferably in a controlled situation, such as the operating room. Airway obstruction is a common cause of death in childhood. Round foods (hot dogs, peanuts, grapes, popcorn) are the common foreign bodies. Objects that pass below the subglottic space will usually lodge in the bronchus and due to its more vertical position, the right main stem bronchus is more common than the left. Larger objects lodge in the upper airway and trachea causing more obvious symptoms of dyspnea, stridor, drooling and cyanosis. These objects can cause partial or complete obstruction and may change positions if the patient changes position. It is important to try not to agitate the patient and to keep them in the position that is most comfortable for them. Position change can easily convert a partial obstruction into a complete one. If the patient is able to maintain their ventilation and oxygenation, the clinician should prepare a "double set-up" with both direct laryngoscopy and needle cricothyrotomy equipment at the bedside. The patient should then be kept still while waiting for back up from ENT, anesthesia or surgery as the safest way to remove the foreign body is with fiberoptic airway devices and possibly in the OR. Back blows (A) are useful in choking infants less than 1 year of age. Moving the patient to a supine position (C) may lead to a complete obstruction. If the patient becomes unconscious, CPR should be initiated. Needle cricothyrotomy (D) may be necessary in the near future but would not be indicated in a patient who is spontaneously breathing and maintaining their oxygenation and ventilation.

3-month-old male infant is rushed to the ED after becoming cyanotic during a prolonged crying spell. His mother says he has had similar episodes in the past and that his pediatrician has told her that he has a loud murmur and will require surgery. On this examination, a faint murmur is heard at the left sternal border, and he is breathing rapidly and heavily. Which of the following pairs is a treatment for this condition with its therapeutic mechanism? Intravenous fluid; decrease pulmonary blood flow Knee-to-chest position; decrease right-to-left shunt Morphine; increase systemic vasodilation Phenylephrine; increase preload

Correct Answer ( B ) Explanation: This is a child with tetralogy of Fallot (TOF) undergoing a hypercyanotic "tet" spell. TOF is characterized by 4 components: (1) right ventricular outflow tract (RVOT) obstruction, (2) right ventricular hypertrophy, (3) ventricular septal defect (VSD), and (4) overriding aorta. During agitation (e.g., crying episodes), there is a lowering of systemic vascular resistance (SVR), which produces a large right-to-left shunt across the VSD, increasing the PCO2 but decreasing the PaO2 and pH. This leads to hyperpnea (deep and rapid breathing), which increases the negative intrathoracic pressure thereby increasing systemic venous return to the right side of the heart. This may worsen the shunt through the VSD and perpetuate a cycle of hypoxia and cyanosis. Commonly, infants with TOF have harsh, loud systolic murmurs at the sternal border, which decrease in intensity during tet spells as blood is shunted away from the RVOT obstruction (often pulmonary stenosis) and through the VSD. To break the cycle of hypoxia, attempts are made to increase SVR and decrease the right-to-left shunt. One of the mainstays in treatment is to place an infant on the abdomen in the knee-to-chest position, which increases SVR. Other treatments include intravenous fluid (A) to increase, rather than decrease, pulmonary blood flow; morphine (C) to reduce the respiratory drive (increasing systemic vasodilation is an undesired side effect); and phenylephrine (D) to increase afterload (SVR) rather than increase preload. Of note, oxygen should be applied to these patients but will not adequately treat tet spells on their own.

1-day-old girl presents to the ED with jaundice, lethargy, and poor tone after a home birth. A total bilirubin returns with a level of 30 mg/dL. This child at risk for developing which of the following complications? Gastric reflux disease Kernicterus Muscular dystrophy Necrotizing enterocolitis

Correct Answer ( B ) Explanation: This patient is exhibiting signs and symptoms consistent with severe neonatal jaundice. Jaundice is defined as discoloration of the skin and sclera caused by the accumulation of unconjugated or conjugated bilirubin. Jaundice on the first day of life is frequently caused by blood group system or ABO incompatibility. Severe jaundice with a total bilirubin > 25 mg/dL places this patient at risk for kernicterus, which is defined as neurotoxic hyperbilirubinemia. Kernicterus presents in phases. The first phase, seen on the first few days of life, is characterized by lethargy, hypotonia, and poor feeding. Phase 2 is characterized specifically by hypotonia of the extensor muscles. Phase 3, which is seen with infants over 1 week of age with persistent severe jaundice, is characterized by diffuse severe hypotonia. Chronic bilirubin encephalopathy can lead to persistent hypotonia, hyperreflexia, delayed motor milestones, and variable cognitive deficits. MRI will classically reveal bilateral symmetric high-intensity signals in the globus pallidus in patients with kernicterus. Gastric reflux disease (A) is not related to jaundice or elevated bilirubin levels. Necrotizing enterocolitis (D) will present with feeding intolerance and abdominal distension instead of jaundice. Abdominal radiographs reveal pneumatosis intestinalis. Muscular dystrophy (C) is a genetic disorder characterized by progressive muscle weakness, most often seen in the proximal lower extremities first. It is not associated with jaundice.

3-year-old boy presents with abdominal pain and vomiting. Mom states that for the last 2 days, her son has intermittently complained of severe abdominal pain and vomiting. Over the last 2 hours, the pain has become constant, and the vomitus has become bilious. You do not have access to ultrasound. Which of the following is the most definitive test for the suspected condition? Abdominal X-ray Air or barium contrast enema CT scan of the abdomen and pelvis with contrast Esophagogastroduodenoscopy

Correct Answer ( B ) Explanation: This patient is suffering from intussusception. Intussusception is a process in which a segment of the intestine invaginates into the adjoining intestinal lumen. This invagination can cause a bowel obstruction. The classic triad for intussusception is abdominal pain, vomiting, and hematochezia. Intussusception is seen mainly in children between 5 months and 3 years of age. Abdominal pain is often intermittent and severe and accompanied by nonbilious vomiting. In between episodes, children are usually well appearing. Once intestinal obstruction occurs, pain is constant and vomitus becomes bilious. A contrast enema with either air or barium can be both diagnostic and therapeutic. Surgery is required if contrast enema does not reduce the intussusception. Ultrasound is now the preferred initial diagnostic study for intussusception. One study reported that the overall sensitivity and specificity of ultrasonography for detecting ileocolic intussusception was 97.9% and 97.8%, respectively. Ultrasonography eliminates the risk of exposure to ionizing radiation and can help to depict lead points and residual intussusceptions. It also helps to rule out other possible causes of abdominal pain. However, not all institutions have 24 hour a day ultrasound available. CT scan (C) can only be diagnostic, exposes the patient to a significant amount of radiation, and does not provide any significantly better data than ultrasound. Abdominal X-ray (A) only reveals signs suggestive of intussusception in 60% of cases. Esophagogastroduodenoscopy (D) does not play a role in the diagnosis or treatment of intussusception. There is an increased risk of perforation with esophagogastroduodenoscopy.

Which of the following is an acyanotic congenital heart defect? Hypoplastic left heart syndrome Isolated ventricular septal defect Tetralogy of Fallot Tricuspid atresia

Correct Answer ( B ) Explanation: Ventricular septal defects (VSD) are the most common congenital heart defects (CHD). Following delivery, there is a decrease in the pulmonary vascular resistance resulting in the left ventricle contracting against relatively higher (systemic) vascular resistance than the right ventricle (pulmonary). This difference leads to a left-to-right shunt in the setting of a VSD. Left-to-right shunts provide increased pulmonary blood flow and oxygenation. They typically do not lead to cyanosis. In contrast, right-to-left shunts lead to decreased pulmonary blood flow and oxygenation with a resultant relative increase in deoxygenated blood in the systemic system. This is a cause of cyanosis. Hypoplastic left heart syndrome (A), tetralogy of Fallot (C), and tricuspid atresia (D) all feature right-to-left shunts.

A four-year-old child presents to the emergency room with high fever, barking cough, stridor and moderate retractions at rest. Which of the following is the most appropriate treatment for this condition? Albuterol nebulizer Ceftriaxone Nebulized racemic epinephrine Oral oseltamivir

Correct Answer ( C ) Explanation: A child with moderate croup has a barking cough, stridor, and moderate retractions at rest. Nebulized epinephrine is the treatment of choice as the rapid local vasoconstriction of the subglottic mucosa reduces swelling and provides rapid improvement. Oral or intravenous steroids should be administered as well to prevent the progression or airway edema as well as rebound edema that may occur up to three hours after the administration of racemic epinephrine. Humidified oxygen can also be given to help control airway edema. Evidence is poor, but in severe cases, children may benefit from heliox. This is a mixture of helium and oxygen which is less dense than air. It assists in air flow through a narrowed airway. In patients that require intubation, an uncuffed endoctracheal tube at least 0.5 mm narrower than the standard for age should be used given concern for significant airway swelling. Albuterol nebulizers (A) should be given in patients with concern for reactive airway disease. It is not indicated in patients with concern for upper airway obstruction. Ceftriaxone (B) is a third-generation cephalosporin that is not indicated in the treatment of viral croup. Oseltamivir (D) is used in the treatment of influenza virus.

A four-year-old boy is brought to the ED for right hip pain for the last three days. He has been walking with a slight limp. On examination, he has minimal pain with passive movement of the right hip. Which of the following is most consistent with a diagnosis of transient synovitis rather than septic arthritis? AAbnormal radiographs of the hip BErythrocyte sedimentation rate of 50 mm/hr COral temperature of 36.8°C DSerum white blood cell count of 20,000 cells/µL

Correct Answer ( C ) Explanation: An oral temperature of 36.8°C is most consistent with a diagnosis of transient synovitis, as generally children with transient synovitis are afebrile. The most important part of diagnosing transient synovitis is to exclude underlying septic arthritis. Transient synovitis is a self-limited non-pyogenic inflammation of the synovium that most commonly occurs in boys between the ages of three and six years of age. The exact etiology is unknown although in many cases follows a mild upper respiratory infection. Children will complain of hip pain, walk with a limp or refuse to bear weight. They are generally otherwise well appearing and non-toxic. This is a diagnosis of exclusion and if there is any concern for septic arthritis, the patient should undergo arthrocentesis and synovial fluid analysis to exclude septic arthritis. Patients with transient synovitis often respond to non-steroidal anti-inflammatory medications and rest. They can generally be managed at home with 12-24 hour follow up at the primary care physician's office. The prognosis is excellent. Radiographs should be obtained in cases of suspected transient synovitis to rule out other forms of pathology, as radiographs (A) of the hip are generally normal in cases of transient synovitis. The erythrocyte sedimentation rate (B) and serum white blood cell count (D) are usually normal in cases of transient synovitis and elevations support a diagnosis of septic arthritis.

2-year-old previously healthy female presents to the ED with complaints of 3 days of low-grade fever and congestion followed by noisy breathing and cough. Upon examination, you note a frequent barking cough, audible stridor at rest, and retractions. The child does not appear to be in any distress. Which of the following is the most appropriate next step in management? The patient has mild croup and should be discharged after single dose of steroid and 5 mg albuterol nebulizer The patient has moderate croup and should receive oral steroids and be discharged after racemic epinephrine nebulizer with next-day follow-up The patient has moderate croup and should receive oral steroids and racemic epinephrine nebulizer and be observed in the ED for 3 hours and discharged if symptoms abate This patient has mild croup and can be discharged after a single dose of oral steroids

Correct Answer ( C ) Explanation: Croup (laryngotracheitis) is the most common cause of infectious acute upper airway obstruction (stridor). The etiology is viral (parainfluenza, influenza, and RSV) with erythema and swelling of the trachea just below the vocal cords. Patients classically present with a barky or seal-like cough. The mean age of affected patients is 18 months. There is a seasonal increase in autumn and early winter. Because the lungs are not directly affected, oxygen saturation can be maintained, even in severe illness. Aerosolized epinephrine (R or L) decreases airway obstruction. It is indicated for children with stridor at rest or marked work of breathing (tachypnea, retractions, accessory muscle use). Maximal effect is seen within 30 minutes, with potential rebound to baseline within 3 hours. Patients without resting stridor after 3 hours can be safely discharged home. A single dose of oral dexamethasone decreases the need for hospitalization and return ED visits. The patient's croup is moderate, not mild (A and D); therefore, she should receive aerosolized epinephrine to treat the airway obstruction. Albuterol should not be used for croup because stimulation of vascular beta-receptors in the airway may cause vasodilation and worsen airway edema. Mild croup is managed with single-dose steroid, and the patient can usually be safely discharged home. It is prudent to observe patients for at least 3 hours if they receive aerosolized epinephrine (B) because symptoms may recur within this period. Some studies have found that rebound occurred in over 30% of cases after the 2nd hour. If symptoms reoccur, another aerosolized epinephrine treatment is indicated and the patient should be admitted.

18-month-old boy presents with his father for a cough and difficulty breathing. The father states that the child has had nasal congestion and coryza for the last two days. On exam, he has a barking cough, inspiratory stridor, and a prolonged inspiratory phase. Which of the following would you expect to see on radiographic evaluation? Edema of the epiglottis Intraluminal membranes of the tracheal wall Subglottic narrowing Thickening of the retropharyngeal space

Correct Answer ( C ) Explanation: Subglottic narrowing and distention of the hypopharynx may be seen on chest radiograph or a soft tissue neck radiograph, respectively, in a child with croup. Subglottic narrowing is often called the "steeple sign" since on the posterior to anterior view of the chest, the trachea appears to narrow toward the top and come to a point like a steeple. Croup is typically caused by parainfluenza virus although a number of other viruses as well as mycobacterial pneumonia can also cause croup. Croup most often occurs in children and is less likely after six years of age. The virus causes swelling just below the vocal folds in the subglottic area. This results in the characteristic "barking" cough and inspiratory stridor. The onset of illness usually begins with several days of nasal congestion and coryza. A fever may also be present. Care should be taken to distinguish cases of croup from epiglottitis or other causes of acute upper airway obstruction. This can be accomplished through careful history taking, physical exam, and radiographic findings. Treatment should initially begin with dexamethasone and possibly nebulized epinephrine depending on severity. Children who are observed for a period of time and do not have recurrent or persistent symptoms, have good air exchange, and normal vital signs may be discharged home. For children with persistent, severe symptoms, hospitalization is indicated. Edema of the epiglottis (A) is seen in epiglottitis and can be noted on a lateral soft tissue radiograph of the neck as the "thumb print" sign. Epiglottitis can quickly cause airway compromise and if this finding is present, steps should be taken to protect the airway. Visualization of intraluminal membranes of the tracheal wall (B) is a finding with bacterial tracheitis. Patients with bacterial tracheitis may initially present with fever, cough, and stridor but rapidly progress to upper airway obstruction. They are often toxic-appearing. Thickening of the retropharyngeal space (D) can be seen with a retropharyngeal abscess. Symptoms include pain in the throat, fever, difficulty swallowing or breathing, and neck stiffness.

What is the most common cause of croup? Adenovirus Haemophilus influenzae type b Parainfluenza virus Streptococcus spp.

Correct Answer ( C ) Explanation: The parainfluenza virus is the most common cause of croup. Croup is classically associated with a barking, seal-like cough and inspiratory stridor. Radiographs may show subglottic narrowing ("steeple sign") caused by edema. The typical age group is 6 months to 3 years, but the condition can be seen in children up to 5 years. The infection and inflammation are usually self-limiting, and conservative management is recommended. Evidence supports the routine use of corticosteroids in most children with croup. Intervention at an earlier phase of the illness reduces the severity of symptoms and the rates of return to a health care practitioner for additional medical attention, ED visits, and hospital admissions. Many children respond to a single, oral dose of dexamethasone. For those who do not tolerate the oral preparation, nebulized budesonide or intramuscular dexamethasone are reasonable alternatives. Mild disease can be treated with humidified oxygen. Moderate to severe disease should be treated with steroids and nebulized racemic epinephrine. Adenovirus (A) is a common cause of URIs, but less common than parainfluenza virus as a cause for croup. Haemophilus influenzae type b (B) is implicated in various infections such as otitis media and epiglottitis, but less common as a cause of croup. Streptococcus spp. (D) infection is not a common cause of croup.

3-year-old boy presents to the ED for difficulty breathing. His parents report that whenever he has a cold he wheezes and it has improved with an albuterol nebulizer treatment that his pediatrician had prescribed. He has been coughing with rhinorrhea for the past few days. When he started wheezing last night, his parents gave him the albuterol treatment every 4 hours. However, this morning he continues to have increased work of breathing despite another albuterol treatment. His vital signs are T 37.2°C, HR 95, RR 28, and a pulse oximetry of 89%. On exam, he is lying in mom's arms quietly with nasal flaring, supraclavicular and intercostal retractions, no wheezing, and minimal air entry auscultated bilaterally. You start inhaled ipratropium and continuous nebulized albuterol and give a dose of corticosteroid. He continues to have severe respiratory distress. What is the next best step in management? AAdminister inhaled budesonide BAdminister intravenous aminophylline CAdminister intravenous magnesium sulfate DAdminister oral montelukast

Correct Answer ( C ) Explanation: The patient in this vignette is in status asthmaticus, defined as a prolonged and severe asthma attack that does not respond to standard treatment. Asthma is the most common chronic disease in childhood, especially in children under 4 years of age, and is more common in males than in females. Inhaled beta agonists such as albuterol are first-line therapies and may be coadministered with inhaled anticholinergic agents such as ipratropium for moderate-to-severe asthma exacerbation. For severe asthmatics, continuous nebulized albuterol may be more effective, but it has the disadvantages of decreased assessment of patient, tachycardia, jitteriness, and hypokalemia. Corticosteroids are also a mainstay of asthma therapy but require hours to be effective. For children presenting with severe asthma exacerbation who are not responding to initial therapies, intravenous magnesium sulfate may be a therapeutic option. Magnesium is used as a smooth muscle relaxant by acting on calcium channels to competitively block calcium influx and is thought to promote bronchodilation. Although a definite therapeutic benefit has yet to be proven, several studies have suggested it may decrease admissions and improve lung function for children with severe exacerbations. Magnesium is generally well tolerated at 25-75 mg/kg over 20 minutes and has a good safety profile with minimal side effects. Rarely hypotension can result secondary to the smooth muscle relaxation effect and must be monitored closely when administered. Furthermore, a patient with status asthmaticus is predisposed to dehydration and intravascular volume depletion from respiratory losses and as a side effect of beta agonists. Therefore, fluid loading is important to help blunt some of the tachycardia associated with treatment agents. Status asthmaticus patients may also benefit from subcutaneous epinephrine or terbutaline, and noninva Theophylline or its intravenous form, aminophylline (B), was once commonly used as maintenance therapy for asthma. However, aminophylline has not shown any benefit for children with mild or moderate asthma exacerbations, even among those requiring hospitalization. In critically ill children unresponsive to other therapies, aminophylline may improve lung function over several hours to days but has not been shown to acutely reduce symptoms, avoid intubation or intensive care unit admission, or decrease length of stay. In addition, it has high incidence of toxicity and a narrow therapeutic range requiring repeated assessment of drug levels. It may result in vomiting, headache, abdominal pain, and palpitations and offers no benefits in the acute emergent management of asthma. Inhaled budesonide (A) and fluticasone are inhaled corticosteroids used as a daily controller therapy rather than for an acute exacerbation. As mediators of inflammation, leukotrienes are minimally affected by corticosteroids, and the addition of oral leukotriene receptor antagonists (LTRAs) such as montelukast (D) to daily asthma treatment can improve lung function and reduce symptoms and exacerbations for many children. However, it is unclear if there are benefits of using LTRAs in the acute setting.

24-month-old boy presents to the ED with cough and noisy breathing. Per mom, he awoke with a barking cough and seemed to be having trouble breathing. On physical exam, the child has stridor at rest and a coarse barking cough. Which of the following is the most common causative organism of this disease process? Coxsackievirus A Haemophilus influenzae type B Parainfluenza type 1 Streptococcus pneumoniae

Correct Answer ( C ) Explanation: This child is exhibiting signs and symptoms consistent with croup, also known as laryngotracheobronchitis. The most common causative agent of croup is parainfluenza virus type 1. Croup primarily affects children ages 6 to 36 months. It is most commonly seen in the fall and winter months. Signs and symptoms of croup include a prodrome of upper respiratory tract infection symptoms for one to three days followed by the development of the characteristic barking seal-like cough and stridor. Children may also present with respiratory distress and retractions. Symptoms are classically worse at night and may improve with taking the child outside into the cool night air. Diagnosis of croup is clinical; however, if there is diagnostic uncertainty, a chest or neck radiograph will likely show a steeple sign caused by subglottic edema and narrowing. Management of patients with severe croup, most often defined as stridor at rest, should be treated with racemic epinephrine. If racemic epinephrine is administered, the child should be observed for two to three hours after treatment to ensure there is no rebound of severe symptoms. Management of all croup patients includes steroids regardless of symptom severity. Children with croup should be admitted to the hospital if they have continued stridor after racemic epinephrine, if stridor returns during the four-hour observation period, or if they require repeated dosing of racemic epinephrine

4-year-old girl presents to the ED after one episode of generalized convulsions during which she was unresponsive. Parents note that she was confused afterward. Her temperature in the ED is 101.4°F. Which of the following is considered diagnostic criteria for a simple febrile seizure? Duration 15 to 20 minutes Localizing neurologic exam No recurrence within 24 hours Partial seizure

Correct Answer ( C ) Explanation: This child is presenting with signs and symptoms consistent with a simple febrile seizure. Febrile seizures are defined as seizures associated with fever in children age 6 months to 5 years. Febrile seizures are a diagnosis of exclusion. Simple febrile seizures represent 85% of all febrile seizures. A simple febrile seizure is characterized by the following criteria: (1) generalized tonic-clonic seizure, (2) occurring in the appropriate age group of age 6 months to 5 years, (3) seizure is less than 15 minutes in duration, (4) the child has a nonfocal neurologic exam, and (5) there is no recurrence of seizures within 24 hours. A complex febrile seizure, representing the remaining 15% of febrile seizures, is any event not meeting the above criteria for a simple febrile seizure. Management of simple febrile seizures is supportive. An infectious etiology should be sought on physical exam and by history taking. Anitpyretics have not been shown to decrease frequency or prevent recurrence of febrile seizures.

9-month-old boy presents to the ED with lethargy and bilious emesis. On physical exam, the child is sleepy and has a very distended abdomen. His abdominal X-ray is shown above. Which of the following is the most likely diagnosis? Hirschsprung disease Intussusception Midgut volvulus Pyloric stenosis

Correct Answer ( C ) Explanation: This child is presenting with signs, symptoms, and imaging findings consistent with malrotation and midgut volvulus. The etiology of malrotation is incomplete rotation of the intestine around the superior mesenteric artery (SMA). This happens secondary to abnormal attachment of the mesentery to the abdominal wall and ligament of Treitz. In the end, malrotation causes volvulus, which occurs when the SMA is strangled by the intestine and causes midgut ischemia. Patients present with midgut volvulus 75% of the time before the age of one year. Signs and symptoms of malrotation with midgut volvulus are that of complete small bowel obstruction and include bilious emesis, abdominal pain, and abdominal distension. Blood in the stool is a late finding, suggesting gut ischemia. Sepsis and shock may occur with necrotic bowel. Diagnosis of malrotation with midgut volvulus is made on upper GI series, which reveals a "corkscrew" appearance of the contrast. Abdominal X-ray may show a "double-bubble" sign, but it is not a definitive study as this may also be seen in duodenal atresia. Management of malrotation with midgut volvulus is with fluid resuscitation, nasogastric tube decompression, and definitively with surgical correction. Hirschsprung disease (A) typically presents with lack of passage of meconium or severe constipation. Intussusception (B) typically presents with intermittent abdominal pain and nonbilious vomiting. Currant jelly stool is a late finding. Pyloric stenosis (D) presents with projectile nonbilious vomiting with immediate hunger afterward. None of these disease characteristically presents with bilious emesis.

A neonate presents to the ED with purulent conjunctivitis with profuse exudate and swelling of the eyelids. Gram stain of the drainage reveals gram negative diplococci. What is the most common age of presentation for this disease process? 14 to 28 days 2 to 14 days 2 to 5 days 5 to 14 days

Correct Answer ( C ) Explanation: This patient is presenting with signs and gram stain findings consistent with gonorrheal neonatal conjunctivitis. Gonorrheal neonatal conjunctivitis typically presents two to five days after birth. It presents with bilateral copious purulent discharge and is often accompanied by systemic symptoms. It is potentially life-threatening. Diagnosis of gonorrheal neonatal conjunctivitis is made on gram stain as it reveals gram negative diplococci. A full sepsis workup should also be initiated due to the high risk of systemic disease. Septic arthritis is the most common manifestation of disseminated disease in this age group. Management of gonorrheal neonatal conjunctivitis is with intravenous cefotaxime and admission. The eyes should also be frequently and copiously irrigated with saline.

3-day-old boy presents to the ED with his parents for bilateral eye drainage. On physical exam, you note purulent conjunctivitis, with profuse exudate and swelling of the eyelids. Which of the following organisms is the most likely etiology of this patient's presentation? Chlamydia trachomatis Herpes simplex virus Neisseria gonorrhoeae Respiratory syncytial virus

Correct Answer ( C ) Explanation: This patient is presenting with signs and symptoms consistent with gonorrheal neonatal conjunctivitis. The causative organism of this disease process is Neisseria gonorrhoeae. Gonorrheal neonatal conjunctivitis typically presents two to five days after birth. It presents with bilateral copious purulent discharge, often swelling of the eyelids and is frequently accompanied by systemic symptoms. It is potentially life-threatening. Diagnosis of gonorrheal neonatal conjunctivitis is made on gram stain as it reveals gram negative diplococci. A full sepsis workup should also be initiated due to the high risk of systemic disease. Management of gonorrheal neonatal conjunctivitis is with intravenous cefotaxime and admission. The eyes should also be frequently and copiously irrigated with saline. Chlamydia trachomatis (A) is a common cause of neonatal conjunctivitis, but it typically presents five to 14 days after birth with unilateral ocular hyperemia, conjunctival edema, and unilateral watery or purulent discharge. Herpes simplex virus (B) can cause neonatal conjunctivitis. It typically presents within two to 14 days after birth. It presents with an infected, red, painful eye, and the diagnosis is confirmed by the presence of epithelial dendrites on slit lamp exam. Respiratory syncytial virus (D) is a common cause of viral conjunctivitis in children outside the neonatal period.

An ex-35-week baby girl presents to the ED two weeks after birth with poor feeding, abdominal distension, and vomiting. Abdominal X-ray is shown above. Which of the following is the most likely diagnosis? Intussusception Midgut volvulus Necrotizing enterocolitis Pyloric stenosis

Correct Answer ( C ) Explanation: This patient is presenting with signs and symptoms consistent with necrotizing enterocolitis (NEC). NEC is defined as bowel necrosis of an unknown etiology. It classically affects premature infants at a much higher rate than those born at term. Signs and symptoms of NEC include abdominal distension, poor feeding, vomiting, and bloody stool. Abdominal wall erythema seen on physical exam is a rare, but pathognomonic finding of NEC. Diagnosis of NEC is with abdominal radiograph, which reveals pneumatosis intestinalis (intramural air within the bowel wall). This radiographic finding is pathognomonic for NEC. Management is first with prolonged bowel rest, nasogastric decompression, and antibiotics. Intussusception (A) presents with vomiting and colicky severe abdominal pain. A late finding of intussusception is currant jelly stool. Midgut volvulus (B) presents with bilious emesis. Pyloric stenosis (D) presents with projectile nonbilious vomiting after feeding followed by immediate hunger.

4-year-old boy without vaccinations presents with a sore throat. Mom has also noted that the patient's voice sounds muffled. On physical examination, the patient is ill-appearing, sitting forward with some drooling. What is the most appropriate next step? Insert two intravenous lines Inspect the posterior pharynx using a tongue depressor Move the patient to the operating room for evaluation with ENT Racemic epinephrine nebulizer

Correct Answer ( C ) Explanation: This patient presents with epiglottitis. Epiglottitis has become less common in children since the development of the H. influenzae vaccine. Classically these children are toxic appearing with significant pain in the throat or neck or both. In severe cases, patients will sit in the sniffing position, often "tripoding" in order to maintain an adequate airway. Given the degree of epiglottal inflammation, patients are at high risk for airway obstruction and all efforts to avoid agitating the patient should be attempted. Since the illness is rapidly progressive, patients will often need definitive airway management. In order to maximize the success of this, management in the operating room with resources and a controlled environment is indicated. Additionally, surgeons are present if the need arises for a surgical airway. In addition to airway management, patients need antibiotic therapy, most commonly ceftriaxone. The insertion of two intravenous lines (A) will likely agitate this child. With increased crying, additional compromise of the tenuous airway will take place and should be avoided at all costs. Access is important in this life-threatening emergency, but optimizing insertion is important (in the OR if time permits or one line to begin). Although it may be helpful to identify swelling in the posterior pharynx, inspection of the posterior pharynx using a tongue depressor (B) is not indicated. This will also agitate the child and further exacerbate the already compromised airway. Racemic epinephrine nebulizers (D) are indicated in cases of croup where tracheal edema causes stridor. Racemic epinephrine is not routine for the management of epiglottitis and again a mask over the child's face may upset him and cause additional difficulty breathing.

A three-year-old girl with a past medical history of acute lymphoblastic leukemia presents for evaluation of altered mental status. Her father reports she started chemotherapy two days ago. During your examination, the patient becomes unresponsive and has a seizure. Which of the following lab abnormality is most likely to be identified in this patient? Hypercalcemia Hypernatremia Hyperphosphatemia Hypokalemia

Correct Answer ( C ) Explanation: This patient's presentation is most concerning for tumor lysis syndrome. This syndrome is most commonly associated with recent chemotherapy for hematologic malignancies. Presenting symptoms are due to the underlying electrolyte abnormalities caused by rapid breakdown of tumor cells. Cell lysis releases tumor DNA and potassium, leading to hyperkalemia. The phosphorus concentration in malignant cells is up to four times higher than in normal cells. Thus, rapid tumor breakdown leading to degradation of the phosphate backbone and purines that make up the tumor's DNA often leads to hyperphosphatemia, which can cause secondary hypocalcemia, resulting in hyperphosphatemia and hyperuricemia. The released phosphate precipitates with calcium which can cause severe hypocalcemia, likely the underlying cause of this patient's seizure. Emergent treatment of tumor lysis syndrome requires normalization of the underlying electrolyte imbalances and aggressive intravenous hydration. Specific treatment of this patient's seizure includes supplemental calcium. Note that calcium is only administered if the patient presents with seizures or cardiac dysrhythmias. Outside of these cases, additional calcium can lead to metastatic calcification, which can result in significant morbidity and mortality due to its widespread effects on multiple organ systems. Sodium abnormalities, like hypernatremia (B), are not specifically associated with tumor lysis syndrome. Elevated sodium may indicate an additional process and should be evaluated separately. Hyperkalemia, not hypokalemia (D) is seen in tumor lysis syndrome. Hypocalcemia, not hypercalcemia (A) is seen in tumor lysis syndrome.

6-week-old boy is brought to the ED for worsening stridor. The parents state that the stridor began approximately 1 week ago after they noticed some clear nasal discharge. The stridor is worse when the patient is agitated and lying in the supine position. On exam, the patient is afebrile and well appearing, but you note stridor when the child is supine. Which of the following is the most likely diagnosis? Aspirated foreign body Tracheal stenosis Tracheomalacia Viral croup

Correct Answer ( C ) Explanation: Tracheomalacia results from abnormally soft, undeveloped supporting cartilage of the tracheal rings. The stridor worsens with agitation, supine position, and infection. It is the most common cause of chronic stridor in infants. It is rarely associated with respiratory distress, feeding difficulties, or failure to thrive. Most patients experience complete resolution of symptoms by age 2. Asphyxia from foreign body aspiration (A) most commonly occurs in children younger than 3 years. Clinical signs of complete obstruction include cough, severe distress, and cyanosis. Tracheal stenosis (B) is a congenital anomaly that results from complete tracheal rings. Infants have persistent stridor that worsens over age. Viral croup (D) is a common pediatric infection associated with a barky cough, hoarse voice, and high-pitched, inspiratory stridor.

A 9-month-old boy presents to the ED after falling a short distance from the couch to the floor. Other than waving his right wrist, he has no other injuries or complaints. You obtain the radiograph seen above. Which of the following is the most likely diagnosis? Greenstick fracture Salter-Harris Type I fracture Salter-Harris Type II fracture Torus fracture

Correct Answer ( D ) Explanation: Bones of children are softer and more resilient than in adults. This leads to more incomplete fractures. A torus, or buckle fracture, is an incomplete fracture characterized by wrinkling or buckling of the cortex. These fractures often occur at the end of long bones. A deformity should not occur in a torus fracture because the periosteum and cortex are intact on the side of the bone opposite to fracture. Radiographically, these are subtle fractures, and care must be taken to avoid misdiagnosis. A Greenstick fracture (A) is also an incomplete angulated fracture of a long bone. Typically, only one side of the cortex is fractured. This results in a bowing appearance of the bone. A Salter-Harris Type I fracture (B) involves an injury to the physis (growth plate) itself, resulting in an apparent displacement of the epiphysis on radiographs with a widening of the radiolucent area on the radiograph. A Salter-Harris Type II fracture (C) is the most common type of Salter-Harris fracture and occurs through the physis and metaphysis; the epiphysis is not involved in the injury.

An otherwise healthy 21-month-old presents to the ED with 3 days of runny nose. Vital signs are T 37.2°C, RR 29, HR 130, and pulse oxygenation 97% on room air. On exam, you hear minimal expiratory wheezing. There are no retractions and no accessory muscle use. You also note copious mucous in both nares. Which of the following is the most appropriate next step in management? Albuterol nebulizer Antibiotics Chest radiograph Reassurance

Correct Answer ( D ) Explanation: Bronchiolitis is a respiratory disease involving the lower respiratory tract that most commonly affects infants between 2 months and 2 years of age. The viral infection results in peribronchiolar cellular infiltrate and mucus plugging, leading to wheezing and fine rales. It is often a self-limited disease managed with supportive care (antipyretics for fever and nasal suctioning for comfort). In a patient with mild disease, good follow-up and no other indications for admission, reassurance is all that is needed. Risk factors associated with severe disease include a history of cyanosis or apnea, age <3 months, atelectasis on chest radiograph, tachypnea with accessory muscle use, and toxic appearance. Infants who appear dehydrated should receive fluid intravenous hydration. Humidified oxygen is also used in supportive management. No strong evidence supports the routine use of beta-agonists (albuterol) (A) in bronchiolitis. Nebulized epinephrine has been shown to have greater clinical benefit than beta-agonists have. Because it is difficult to distinguish asthma from bronchiolitis, it is reasonable to administer albuterol in patients who have atopy, eczema, or a family history of asthma. Antibiotics (B) are not routinely advised in bronchiolitis in the absence of comorbid suspicion for pneumonia (fever, focal findings, leukocytosis). Bronchiolitis is a viral disease, most commonly caused by the respiratory syncytial virus (RSV). Aerosolized ribavirin should be considered in hospitalized infants. In the absence of focal findings and a well-appearing patient, a chest radiograph (C) does not need to be routinely obtained for the diagnosis of bronchiolitis.

Which of the following statements is true regarding febrile seizures? AAdministering acetaminophen and ibuprofen during a febrile illness have been shown to decrease the likelihood of seizure recurrence BChildren who have had a simple febrile seizure have the same rate of epilepsy as those who have not had a febrile seizure COlder children with a febrile seizure are more likely to have a recurrence than younger children with a febrile seizure are DTreatment with long-term anticonvulsants does not lower the long-term risk of developing epilepsy

Correct Answer ( D ) Explanation: Children with simple febrile seizures have a 2% to 3% chance of developing epilepsy, compared with a 1% rate of epilepsy in the general population. Children with complex febrile seizures have a significantly higher risk. Treatment with long-term anticonvulsants does not affect the long-term risk of developing epilepsy and is rarely warranted. Administering acetaminophen and ibuprofen (A) have not been shown to decrease the likelihood of seizure recurrence. About 30% of children with a simple febrile seizure have a recurrence; of these, one-half will have a third event. Children with simple febrile seizures have a 2% to 3% chance of developing epilepsy (B), compared with a 1% rate of epilepsy in the general population. The younger the age at onset of a febrile seizure, the more likelihood of recurrence (C).

In children under 8-years of age, which of the following is the most common fracture involving the elbow? Monteggia fracture Nursemaid's elbow Radial head fracture Supracondylar fracture

Correct Answer ( D ) Explanation: In children under 8-years of age, the most common fracture involving the elbow is a supracondylar fracture. The mechanism is usually due to a fall on an outstretched hand (FOOSH). There are three types of fractures. Type I fractures are only evident by a displaced anterior fat pad ("sail sign"), presence of a posterior fat pad (which always indicates occult fracture) or displacement of anterior humeral line (which normally intersects the middle third of the capitellum), or a combination of these. Types II and III describe hinge mechanisms or complete displacement and are usually reduced in the ED or OR. Supracondylar fractures are high risk for neurovascular injury to the brachial artery and median nerve. Careful neurovascular checks and prompt treatment are key to prevent complications such as compartment syndrome and Volkmann's ischemic contracture. Monteggia fractures (A) are less common in children and should have forearm radiographs to completely demonstrate the proximal ulnar fracture and radial head dislocation. A child who was swung by the wrist may have a subluxation of the radial head resulting in a nursemaids elbow (B). Radial head fractures (C) are more common in adults. These fractures are also associated with a posterior and anterior fat pad sign.

At what age do pediatric cervical spine injuries begin to follow the same pattern as that of adults? 10 years 12 years 5 years 8 years

Correct Answer ( D ) Explanation: Pediatric cervical spine injuries are relatively uncommon due to the flexibility of the pediatric spine; however, bony cervical spine injuries remain possible in the pediatric population. Children < 8 years are more likely to have a high cervical spine injury due to their relatively large head compared to their body and the relative lack of strength in their neck muscles to support their larger heads. These injuries are more frequently seen in C1 through C3 while adult cervical spine injuries more commonly occur in the lower cervical spine. Pediatric cervical spine injuries begin to follow the adult injury pattern at 8 years of age. At 5 years (C) of age, patients are more likely to have high cervical spine injuries. At 10 (A) and 12 years (B) of age, children are more likely to suffer lower cervical spine injuries similar to adults

1-month-old girl born at 32 weeks gestation is brought to the ED for vomiting. Mom reports that her daughter weighed 2,000 grams when she was born and stayed in the neonatal intensive care unit for 1 week for treatment of hyperbilirubinemia and respiratory distress syndrome. She was discharged home last week and had been tolerating her premature milk-based formula feeds without any issues. However, she developed green-tinged emesis with her last 2 feedings and has not had a bowel movement since yesterday morning. In the ED, her vital signs are T 35.8ºC, BP 70/30, HR 90, RR 20, and pulse oximetry of 100%. Physical exam reveals a listless infant with a distended abdomen and hypoactive bowel sounds. You obtain the radiograph seen above. Which of the following is the most likely diagnosis? Constipation Cow milk protein intolerance Intestinal malrotation Necrotizing enterocolitis

Correct Answer ( D ) Explanation: The infant has pneumatosis intestinalis on the abdominal radiograph, which is the hallmark of necrotizing enterocolitis (NEC). NEC is primarily a disease of the preterm infant. The development of NEC is associated with enteral feeding, particularly with nonhuman milk. NEC often presents several days after the initiation of feedings with nonspecific clinical findings such as temperature instability, apnea, bradycardia, lethargy, and feeding intolerance. NEC is commonly categorized by Bell stages. Stage I is early or suspected NEC based on feeding intolerance, vomiting, or ileus. Stage II is definite NEC proven with abdominal radiographs of pneumatosis intestinalis. Stage III represents advanced disease and is associated with perforation. Constipation (A) is a frequent presenting complaint in neonates. A detailed history and exam are important to differentiate constipation from other life-threatening processes. Subjective associated symptoms such as grunting, straining, or turning red during defecation are normal, though often considered indicative of constipation. Constipation may cause mild abdominal fullness, but significant distension is abnormal. There is occasional abdominal tenderness, usually in the left lower quadrant. Constipation should never cause peritoneal findings. Rectal exam may reveal stool in the rectal vault, which is more consistent with constipation. Absence of stool in the vault is concerning for a more proximal pathology such as Hirschsprung's disease. Cow milk protein intolerance (CMPI) (B) affects 3% of infants under the age of 12 months and is often misdiagnosed as GERD or colic. Most infants outgrow CMPI by 12 months; however, those with IgE-mediated reactions usually continue to be intolerant to cow's milk proteins, and they develop other allergens including, environmental allergens that cause asthmatic symptoms. Clinical manifestations of CMPI include diarrhea, bloody stools, vomiting, feeding refusal, eczema, atopic dermatitis, urticaria, angioedema, allergic rhinitis, coughing, wheezing, failure to thrive, and anaphylaxis. Malrotation with midgut volvulus (C) is most commonly seen in the 1st year of life and presents with bilious vomiting. The radiograph may show partial or complete duodenal obstruction or a gasless abdomen. A classic finding on radiography is the double bubble sign, which represents a distended stomach and duodenum.

A 4-year-old boy presents to the Emergency Department with a muffled voice and fever. On physical exam, he is stridulous at rest. Vital signs are BP 90/60 mm Hg, HR 135 beats/minute, RR 36 breaths/minute, and T 103.1°F. Which of the following is the most common position of comfort in which these patients present? Fetal Left lateral decubitus Supine Tripoding

Correct Answer ( D ) Explanation: This child is exhibiting signs and symptoms of epiglottitis. Signs and symptoms of epiglottitis include rapid onset of fever, dysphagia, dysphonia, and stridor. Children will often present in the tripod position and should not be disrupted or removed from their position of comfort. Diagnosis of epiglottitis is clinical. If the diagnosis is not clear, a lateral soft tissue X-ray of the neck could be obtained and would reveal a "thumbprint" sign. This should be done at the bedside given the risk of losing the patient's airway in radiology. Management of epiglottitis includes a primary focus on the airway which involves keeping the patient in a position of comfort and consultation of otolaryngology and anesthesia for appropriate airway management preferably in the operating room. Intravenous antibiotics, which should include a third-generation cephalosporin, should be initiated after securing the airway so as not to agitate the child and cause airway compromise. Epiglottitis is historically most commonly caused by Haemophilus influenzae type B. The fetal (A) position is the most common position of comfort for patients with peritoneal irritation. Left lateral decubitus (B) is commonly a position of comfort to a patient with a mainstem bronchus foreign body on the left to better aerate their right lung. Supine (C) positioning is a position of comfort for many patients with nausea or diseases with symptoms worsened by movement, such as vertigo.

24-month-old boy presents to the ED after one episode of generalized convulsions during which he was unresponsive. Parents note that he wet his pants and was "dazed" for a while afterward. The child has returned to baseline and is walking around the examination room. There are no focal neurologic findings on exam. Oral temperature in the ED is 103.4 °F. Which of the following is the most likely diagnosis? Complex febrile seizure Complex partial seizure Generalized absence seizures Simple febrile seizure

Correct Answer ( D ) Explanation: This child is presenting with signs and symptoms consistent with a simple febrile seizure. Febrile seizures are defined as seizures associated with fever in children ages six months to five years. Febrile seizures are a diagnosis of exclusion. They occur in 3-4% of the population and do have a familial predisposition. The peak occurrence of febrile seizures is from age 14 to 18 months. There is also a strong association with the disease process of roseola. Simple febrile seizures represent 85% of all febrile seizures. A simple febrile seizure is characterized by the following criteria: (1) generalized tonic-clonic seizure, (2) occurring in the appropriate age group of ages six months to five years, (3) seizure is less than 15 minutes in duration, (4) the child has a nonfocal neurologic exam, and (5) there is no recurrence of seizures within 24 hours. Management of simple febrile seizures is supportive. An infectious etiology should be sought on physical exam and by history taking. Antipyretics have not been shown to decrease frequency or prevent recurrence of febrile seizures. Children who have a febrile seizure have a 30% chance of having another febrile seizure. Risk factors for recurrence include age < one year, seizure occurred with low fever, and complex febrile seizure. The risk of epilepsy in children with febrile seizures is 2-10% versus 1% for the general population. A complex febrile seizure (A) is any febrile seizure not meeting the criteria to be a simple febrile seizure. Complex partial seizures (B) are partial seizures often involving one extremity, one side of the body, or one side of the face occurring often in children with a known seizure disorder or a focus in the brain causing seizure activity such as a mass. Generalized absence seizures (C) are characterized by a period of unconsciousness and a blank stare. These seizures are most common in children. Epileptic activity occurs throughout the entire brain. The child may also exhibit chewing movements, rapid breathing, rhythmic blinking, or tugging at the clothing but there is no overall shaking of the extremities.

5-year-old girl presents to the ED with a rash that started on her face and spread to her neck, axillae, and groin. Mom states that the patient had an upper respiratory infection one week prior. On examination, the patient's rash is tender to the touch. Which of the following statements regarding the diagnosis of this patient's condition is correct? Deep layers of the dermis are involved It often leaves the patient disfigured from scarring Mucous membrane involvement is common The disease is caused by an exotoxin-producing bacterium

Correct Answer ( D ) Explanation: This patient has staphylococcal scalded skin syndrome (SSSS), which is caused by an exotoxin-producing strain of Staphylococcus aureus. SSSS is actually a severe form of bullous impetigo. It is usually seen in children < 5 years of age and often follows a upper respiratory infection. The rash usually starts on the face (perioral is classic), neck, axillae, and groin. The rash then spreads, becomes exfoliative, and is followed by the development of flaccid bullae and skin desquamation. The rash is characteristically tender to the touch, and gentle traction on the affected skin produces peeling (positive Nikolsky sign). The cleavage plane is intraepidermal, therefore, only superficial layers of the epidermis (A) shed, and it usually resolve in two weeks without scarring (B). Unlike Stevens-Johnson syndrome, mucous membranes (C) are not involved in SSSS.

2-year-old girl with Pierre Robin syndrome presents with inspiratory stridor and two days of fever. Her respiratory rate is 35/min, temperature is 39.4ºC, heart rate is 135/min, and oxygen saturation is 93% on room air. There are moderate suprasternal and intercostal retractions and you note a barking cough. Which of the following is the most appropriate next step in management? Administer intramuscular epinephrine, diphenhydramine, and dexamethasone Consult an otolaryngologist Defer the examination until you bring the patient to the operating room Place the patient on a facemask with racemic epinephrine

Correct Answer ( D ) Explanation: This patient is presenting with respiratory distress in the setting of probable croup. In addition, she has Pierre Robin syndrome, which is characterized by micrognathia, glossoptosis, and the potential for upper airway obstruction. These patients often have stridor at baseline. In this case, it is reasonable to assume that the patient's stridor is secondary to her infection. Given that croup is a reversible condition, it is appropriate to give a trial of racemic epinephrine. Dexamethasone is indicated in the treatment of croup as well but has a slower onset of action than racemic epinephrine. It should be given early in the course of treatment in cases of moderate to severe croup. If her respiratory status does not improve despite medical management, she may require intubation by an experienced clinician. Deferring some aspects of the physical exam (C) to a safer environment, such as the operating room, may be prudent in an ill-appearing patient with high suspicion for epiglottitis. Nonetheless, you should still attempt temporizing measures. Consulting an otolaryngologist (B) will delay management of this patient. Treatment with racemic epinephrine should be started immediately. Though there is some crossover with croup treatment, intramuscular epinephrine, diphenhydramine, and dexamethasone (A) are specifically indicated for the management of anaphylaxis, a different cause of stridor than the one presented in this case vignette.

A three-day-old girl presents to the ED with choking and "turning blue." Parents note that she seems "normal" unless she is eating, during which she will choke, froth at the mouth, and turn blue in her lips. Physical exam is unremarkable. Which of the following is the most likely diagnosis? Duodenal atresia Midgut volvulus Pyloric stenosis Tracheoesophageal fistula

Correct Answer ( D ) Explanation: This patient is presenting with signs and symptoms consistent with tracheoesophageal fistula (TEF). TEF is a congenital or acquired communication between the trachea and esophagus. Eighty-five percent of cases occur due to proximal esophageal atresia and a distal anastomosis between the trachea and the lower esophagus, distal to the blind proximal pouch. Patients with TEF most commonly present early in the neonatal period. Signs and symptoms of TEF include copious frothy secretions and choking and cyanosis with feeding. Diagnosis of TEF is usually via attempting nasogastric tube placement and encountering early resistance and inability to pass the tube. Chest and abdominal radiographs will reveal a paucity of intestinal bowel gas and coiling of nasogastric tube in proximal esophagus. Management of TEF is with surgical correction. Duodenal atresia (A) presents within the first few days of life with bilious emesis and requires early surgical correction. Midgut volvulus (B) presents with bilious emesis during the first year of life often secondary to malrotation of the abdominal viscera. Pyloric stenosis (C) presents within the first eight weeks of life with projectile nonbilious emesis with feeding followed immediately by hunger.

A 10-month-old girl presents to the ED with a history of persistent cough and recurrent pneumonia. Mom states that she coughs a lot during and after feeds and has had no improvement with use of ß-agonists or acid blockers. Which of the following is the best initial test to diagnose this patient's condition? Arterial blood gas Barium swallow with decubitus views Chest radiograph Water soluble swallow contrast with decubitus views

Correct Answer ( D ) Explanation: This patient's history is consistent with tracheoesophageal fistula (TEF). The most common TEF discovered outside of the newborn period is the H-type fistula. A water soluble contrast swallow is the diagnostic test of choice as barium provokes an extensive inflammatory reaction in the lungs and hinders surgical exploration. In combination with decubitus views, the fistula tract will be identified. Tracheoesophageal fistula is the most common cause of esophageal obstruction in neonates. During embryonic development, the trachea and esophagus normally separate and develop in a linear fashion. With a TEF, there is abnormal communication between the trachea and esophagus An arterial blood gas (A) would help to determine if an arterial-alveolar gradient exists for shunting problems or issues with diffusion of oxygen, but it would not be helpful in determining this patient's diagnosis. A chest radiograph (C) can help in diagnosing an associated pneumonia but will not assist in determining the etiology of this condition. Barium (B) provokes an extensive inflammatory reaction in the lungs and hinders surgical exploration.


Related study sets

Comp sci quiz compendium 8,9,10,12

View Set

Chapter 18 - Cardiovascular System - Blood

View Set

APES Chapter 1; What is an Environmentally Sustainable Society?

View Set

Unanticipated Problems and Reporting Requirements in Social and Behavioral Research

View Set

Chapter 7: Portable Fire Extinguishers

View Set